Putnam 2018/A2

Συντονιστής: Demetres

Άβαταρ μέλους
Demetres
Γενικός Συντονιστής
Δημοσιεύσεις: 8989
Εγγραφή: Δευ Ιαν 19, 2009 5:16 pm
Τοποθεσία: Λεμεσός/Πύλα
Επικοινωνία:

Putnam 2018/A2

#1

Μη αναγνωσμένη δημοσίευση από Demetres » Τρί Δεκ 04, 2018 4:21 pm

Έστω S_1, S_2, \dots, S_{2^n - 1} τα μη κενά υποσύνολα του\{1, 2, \ldots, n\} με κάποια σειρά, και έστω M ο (2^n - 1) \times (2^n - 1) πίνακας του οποίου το (i, j) στοιχείο είναι το

\displaystyle m_{ij} = \begin{cases} 0 & \text{\gr αν } S_i \cap S_j = \emptyset \\ 1 & \text{\gr αν } S_i \cap S_j \neq \emptyset.\end{cases}

Να υπολογιστεί η ορίζουσα του M.



Λέξεις Κλειδιά:
Απάντηση

Επιστροφή σε “Διαγωνισμοί για φοιτητές”

Μέλη σε σύνδεση

Μέλη σε αυτήν τη Δ. Συζήτηση: Google [Bot] και 11 επισκέπτες